0 Daumen
629 Aufrufe

Hallo, ich bin neu hier und habe eine Frage zu einer Aufgabe.
Entschuldigt bitte schon mal die Formatierung. Da ich mit LateX noch nicht sehr vertraut bin, habe ich versucht die Formeln mithilfe von einfachen Tastatursymbolen auszudrücken.


Aufgabe.jpeg

Text erkannt:

\( F: V=\mathbb{Z}_{7}[t]_{2} \longrightarrow W=\left\{A \in \mathbb{Z}_{7}^{2 \times 2} \mid A\right. \) ist symmetrisch \( \}, a+b t+c t^{2} \longmapsto\left(\begin{array}{cc}3 c & a+b+c \\ a+b+c & 3 a\end{array}\right) \)
Weiter seien die Basen \( \mathcal{B}_{1}=\left(2+3 t+t^{2}, 2+2 t+2 t^{2}, 4 t+2 t^{2}\right) \) und \( \mathcal{B}_{2}=\left(1+t, 3+3 t+4 t^{2}, 3+4 t+6 t^{2}\right) \) von \( V, \) sowie \( \mathcal{C}_{1}=\left(\left(\begin{array}{ll}1 & 2 \\ 2 & 3\end{array}\right),\left(\begin{array}{ll}1 & 6 \\ 6 & 5\end{array}\right),\left(\begin{array}{ll}6 & 6 \\ 6 & 5\end{array}\right)\right) \)
und \( \mathcal{C}_{2}=\left(\left(\begin{array}{ll}4 & 6 \\ 6 & 2\end{array}\right),\left(\begin{array}{ll}0 & 4 \\ 4 & 4\end{array}\right),\left(\begin{array}{ll}6 & 0 \\ 0 & 4\end{array}\right)\right) \) von \( W \) gegeben.
(a) Bestimmen Sie die darstellenden Matrizen \( M_{\mathcal{B}_{1}}^{\mathcal{B}_{2}}\left(\mathrm{id}_{V}\right), M_{\mathcal{C}_{2}}^{\mathcal{C}_{1}}\left(\mathrm{id}_{W}\right), M_{\mathcal{C}_{1}}^{\mathcal{B}_{1}}(F), M_{\mathcal{C}_{1}}^{\mathcal{B}_{2}}(F), M_{\mathcal{C}_{2}}^{\mathcal{B}_{1}}(F) \)
und \( M_{\mathcal{C}_{2}}^{\mathcal{B}_{2}}(F) \) unter Verwendung des verallgemeinerten Basiswechsels von linearen Abbildungen



Die gesamte Aufgabenstellung ist im angehängten Bild zu sehen.
Es geht um den Aufgabenteil a). Dort soll man die angegebenen darstellenden Matrizen berechnen.

Mit der Matrix M B2/B1 (das soll hier B2 über B1 heißen) (idV) hatte ich keine Probleme.

Sei hier S die Standardbasis von V.

Denn es gilt: M B2/B1 (idV) = M S/B1 (idV) * M B2/S (idV).
Außerdem gilt: M S/B1 (idV) = M B1/S (idV)^-1 (M S/B1 (idV) ist M B1/S (idV) invertiert).

Nunn ließen sich M B2/S (idV) und M B1/S (idV)^-1 leicht bestimmen.
M B2/S (idV) durch ablesen (da sich wegen der Identität nichts ändert)und M B1/S (idV)^-1 ebenfalls durch ablesen und dann invertieren mittels Gauß-Verfahren.
Dann habe ich M B1/S (idV)^-1 und M B2/S (idV) multipliziert und das ergab natürlich M B2/B1 (idV). Hier bin ich mir sehr sicher, weil ich die Prode gemacht habe und dort alles stimmig war.

Nun zum eigentlichen Problem. Wie in der Aufgabenstellung zu sehen sind dort für W zwei Basen angegeben, die jeweils aus drei 2x2 Matrizen bestehen. Ich muss nun M C1/C2 (idW) berechnen, wobei ich nicht weiterkomme, weil bei mir für M C2/S (idW) eine nicht-quadratische Matrix rauskommt, die ich ja dann nicht invertierbar ist, da nicht quadratisch.
(Sei S hier im zweiten Beispiel die Standardbasis von W.)

Ich danke allen, die sich Mühe machen sich das ganze durchzulesen und vor allem denen, die konstruktive Vorschläge oder Tipps parat haben.


Problem/Ansatz:

Wenn ich alles richtig verstanden habe, muss gelten:
M C1/C2 (idW) = M S/C2 (idW) * M C1/S (idW) also:
M C1/C2 (idW) = M C2/S (idW)^-1 * M C1/S (idW)

Mein Problem ist aber, dass bei mir für M C2/S (idW) immer eine nicht-quadratische 4x3 Matrix rauskommt.
Ich rechne so, dass ich auf die Matrizen von C2 die Abbildung von W nach W anwende. Dann bekomme ich, der Abbildung entsprechend erstmal drei 2x2 Matrizen heraus. Die Matrizen schreibe ich dann jeweils als Spalten in eine größere Matrix 8das wurde uns im Tutorium genauso gezeigt), die 4x3 groß ist und die M C2/S (idW) entsprechen sollte.

Ich hatte schon die Idee die Basis C2 um eine 2x2 Matrix erweitere, z. B. eine 2x2 Einheitsmatrix, aber wäre es ohne weiteres möglich ohne die gegebene Basis C2 zu verändern?

Ich hoffe jemand kann mir helfen, ich stehe gerade auf dem Schlauch.
Ich wäre für jeden Tipp oder Rat dankbar.
(Vielleicht habe ich das im Tutorium doch falsch verstanden oder mein ANsatz ist ganz falsch?)

Text erkannt:

\( F: V=\mathbb{Z}_{7}[t]_{2} \longrightarrow W=\left\{A \in \mathbb{Z}_{7}^{2 \times 2} \mid A\right. \) ist symmetrisch \( \}, a+b t+c t^{2} \longmapsto\left(\begin{array}{cc}3 c & a+b+c \\ a+b+c & 3 a\end{array}\right) \)
Weiter seien die Basen \( \mathcal{B}_{1}=\left(2+3 t+t^{2}, 2+2 t+2 t^{2}, 4 t+2 t^{2}\right) \) und \( \mathcal{B}_{2}=\left(1+t, 3+3 t+4 t^{2}, 3+4 t+6 t^{2}\right) \) von \( V, \) sowie \( \mathcal{C}_{1}=\left(\left(\begin{array}{ll}1 & 2 \\ 2 & 3\end{array}\right),\left(\begin{array}{ll}1 & 6 \\ 6 & 5\end{array}\right),\left(\begin{array}{ll}6 & 6 \\ 6 & 5\end{array}\right)\right) \)
und \( \mathcal{C}_{2}=\left(\left(\begin{array}{ll}4 & 6 \\ 6 & 2\end{array}\right),\left(\begin{array}{ll}0 & 4 \\ 4 & 4\end{array}\right),\left(\begin{array}{ll}6 & 0 \\ 0 & 4\end{array}\right)\right) \) von \( W \) gegeben.
(a) Bestimmen Sie die darstellenden Matrizen \( M_{\mathcal{B}_{1}}^{\mathcal{B}_{2}}\left(\mathrm{id}_{V}\right), M_{\mathcal{C}_{2}}^{\mathcal{C}_{1}}\left(\mathrm{id}_{W}\right), M_{\mathcal{C}_{1}}^{\mathcal{B}_{1}}(F), M_{\mathcal{C}_{1}}^{\mathcal{B}_{2}}(F), M_{\mathcal{C}_{2}}^{\mathcal{B}_{1}}(F) \)
und \( M_{\mathcal{C}_{2}}^{\mathcal{B}_{2}}(F) \) unter Verwendung des verallgemeinerten Basiswechsels von linearen Abbildungen


Text erkannt:

\( F: V=\mathbb{Z}_{7}[t]_{2} \longrightarrow W=\left\{A \in \mathbb{Z}_{7}^{2 \times 2} \mid A\right. \) ist symmetrisch \( \}, a+b t+c t^{2} \longmapsto\left(\begin{array}{cc}3 c & a+b+c \\ a+b+c & 3 a\end{array}\right) \)
Weiter seien die Basen \( \mathcal{B}_{1}=\left(2+3 t+t^{2}, 2+2 t+2 t^{2}, 4 t+2 t^{2}\right) \) und \( \mathcal{B}_{2}=\left(1+t, 3+3 t+4 t^{2}, 3+4 t+6 t^{2}\right) \) von \( V, \) sowie \( \mathcal{C}_{1}=\left(\left(\begin{array}{ll}1 & 2 \\ 2 & 3\end{array}\right),\left(\begin{array}{ll}1 & 6 \\ 6 & 5\end{array}\right),\left(\begin{array}{ll}6 & 6 \\ 6 & 5\end{array}\right)\right) \)
und \( \mathcal{C}_{2}=\left(\left(\begin{array}{ll}4 & 6 \\ 6 & 2\end{array}\right),\left(\begin{array}{ll}0 & 4 \\ 4 & 4\end{array}\right),\left(\begin{array}{ll}6 & 0 \\ 0 & 4\end{array}\right)\right) \) von \( W \) gegeben.
(a) Bestimmen Sie die darstellenden Matrizen \( M_{\mathcal{B}_{1}}^{\mathcal{B}_{2}}\left(\mathrm{id}_{V}\right), M_{\mathcal{C}_{2}}^{\mathcal{C}_{1}}\left(\mathrm{id}_{W}\right), M_{\mathcal{C}_{1}}^{\mathcal{B}_{1}}(F), M_{\mathcal{C}_{1}}^{\mathcal{B}_{2}}(F), M_{\mathcal{C}_{2}}^{\mathcal{B}_{1}}(F) \)
und \( M_{\mathcal{C}_{2}}^{\mathcal{B}_{2}}(F) \) unter Verwendung des verallgemeinerten Basiswechsels von linearen Abbildungen

Avatar von

1 Antwort

0 Daumen
 
Beste Antwort

\( \mathcal{C}_{1}=\left(\left(\begin{array}{ll}1 & 2 \\ 2 & 3\end{array}\right),\left(\begin{array}{ll}1 & 6 \\ 6 & 5\end{array}\right),\left(\begin{array}{ll}6 & 6 \\ 6 & 5\end{array}\right)\right) \)
und \( \mathcal{C}_{2}=\left(\left(\begin{array}{ll}4 & 6 \\ 6 & 2\end{array}\right),\left(\begin{array}{ll}0 & 4 \\ 4 & 4\end{array}\right),\left(\begin{array}{ll}6 & 0 \\ 0 & 4\end{array}\right)\right) \) von \( W \) gegeben.

Ich muss nun M C1/C2 (idW) berechnen.

Also ist C1 die Basis für die Urbilder und C2 die Basis für die Bilder. Du musst also jede Matrix von C1 erst mal abbilden , weil idW die Abbildung ist, bleibt das also gleich und diese Matrix mit der Basis C2 darstellen in der Form

$$(\begin{array}{ll}1 & 2 \\ 2 & 3\end{array})=x*(\begin{array}{ll}4 & 6 \\ 6 & 2\end{array})+y*(\begin{array}{ll}0 & 4 \\ 4 & 4\end{array})+z*(\begin{array}{ll}6 & 0 \\ 0 & 4\end{array})$$

Und die x,y,z sind dann die erste Spalte der gesuchten Matrix.

Mit der 2. Matrix aus C1 bekommst du entsprechend die 2. Spalte etc.

Ich bekomme dann insgesamt $$M_{\mathcal{C}_{2}}^{\mathcal{C}_{1}}\left(\mathrm{id}_{W}\right)=(\begin{array}{ll}1 & 4&5 \\ 6 & 4&6\\ 3 & 0&4\end{array})$$

Avatar von 288 k 🚀

Danka, das hat mir schon sehr weiter geholfen. Ich hätte da nur noch eine Frage.

Ich würde gerne erstmal die darstellende Matrix M C1/S (idW) berechnen, um versuchen die Formel M C1/C2 (idW) = M C2/S (idW)^-1 * M C1/S (idW)anzuwenden. (M C2/S (idW)^-1 bedeutet M C2/S (idW) invertiert).

Dann wären die Basis \( \mathcal{C}_{1}=\left(\left(\begin{array}{ll}1 & 2 \\ 2 & 3\end{array}\right),\left(\begin{array}{ll}1 & 6 \\ 6 & 5\end{array}\right),\left(\begin{array}{ll}6 & 6 \\ 6 & 5\end{array}\right)\right) \) und die Standardbasis \( \mathcal{S}_{'}=\left(\left(\begin{array}{ll}1 & 0 \\ 0 & 0\end{array}\right),\left(\begin{array}{ll} 0 & 1 \\ 0 & 0\end{array}\right), \left(\begin{array}{ll} 0 & 0 \\ 1 & 0\end{array}\right), \left(\begin{array}{ll}0 & 0 \\ 0 & 1\end{array}\right)\right) \)

Um es dann ausführlich mit der Methode von oben zu machen müsste ich C1 mit der Standardbasis darstellen:

\((\begin{array}{ll}1 & 2 \\ 2 & 3\end{array})=x*(\begin{array}{ll}1 & 0 \\ 0 & 0\end{array})+y*(\begin{array}{ll}0 & 1 \\ 0 & 0\end{array})+z*(\begin{array}{ll}0 & 0 \\ 1 & 0\end{array})+q*(\begin{array}{ll}0 & 0 \\ 0 & 1\end{array})\)

Was für x, y, z und q rauskommt sieht man dann auf den ersten Blick. Die erste Spalte der Matrix MC1/S wäre demnach (1, 2, 2, 3). Wenn ich das Verfahren bis zum Ende für alle drei Matrizen anwende, kommt eine 4x3 Matrix raus, wäre das richtig?

Und diese Matrix wäre dann nicht invertierbar, weil sie nicht quadratisch ist.

Nix mit q. Der Raum der SYMMETRISCHEN 2x2 Matrizen ist 3-dim.

Deshalb haben die Basen auch nur 3 Elemente.

Die Stand.basisi kannst du eh nicht brauchen. Die 2. und 4. Matrix sind nicht symmetrisch.

Danke, dass die Matrizen von W symmetrisch sein müssen, habe ich überlesen.

Was wäre denn theoretisch in diesem Fall die Standardbasis von W?

Würde sie dann nur aus den zwei symmetrischen Matrizen bestehen?

Also: \( \mathcal{S}_{'}=\left(\left(\begin{array}{ll}1 & 0 \\ 0 & 0\end{array}\right),\left(\begin{array}{ll} 0 & 0 \\ 1 & 0\end{array}\right)\right) \)

Aber das kann auch nicht richtig sein, weil der Raum 3-dimensional ist, oder?

Genau, wenn du unbedingt eine einfachere Basis haben willst, nimm diese

\( \mathcal{S}_{'}=\left(\left(\begin{array}{ll}1 & 0 \\ 0 & 0\end{array}\right),\left(\begin{array}{ll} 0 & 0 \\ 0 & 1\end{array}\right),\left(\begin{array}{ll} 0 & 1 \\ 1 & 0\end{array}\right)\right) \)

Aber du kannst es doch gleich so machen, wie ich es beschreiben habe.

Vielen Dank für die schnelle Hilfe!

Ein anderes Problem?

Stell deine Frage

Willkommen bei der Mathelounge! Stell deine Frage einfach und kostenlos

x
Made by a lovely community